User contributions for Rmehtany
23 February 2016
- 19:4419:44, 23 February 2016 diff hist −28 m 2016 AMC 12B Problems/Problem 12 There exists a solution
5 February 2016
- 18:5018:50, 5 February 2016 diff hist +2 m 2016 AMC 12A Problems/Problem 23 →Solution 4: More Calculus
- 18:4818:48, 5 February 2016 diff hist +106 2016 AMC 12A Problems/Problem 23 →Solution 1: Logic
- 18:4618:46, 5 February 2016 diff hist +13 m 2016 AMC 12A Problems/Problem 23 LaTeX reformatting
- 18:4518:45, 5 February 2016 diff hist +788 2016 AMC 12A Problems/Problem 23 →Solution
8 December 2015
- 18:4918:49, 8 December 2015 diff hist +534 N 1967 IMO Problems/Problem 5 Created page with "Take |a1| >= |a2| >= ... >= |a8|. Suppose that |a1|, ... , |ar| are all equal and greater than |ar+1|. Then for sufficiently large n, we can ensure that |as|n < 1/8 |a1|n for ..."
19 February 2014
- 08:4108:41, 19 February 2014 diff hist −13 1970 IMO Problems/Problem 6 No edit summary
- 08:4008:40, 19 February 2014 diff hist +799 1970 IMO Problems/Problem 6 No edit summary